Difference between revisions of "2021 AMC 12A Problems/Problem 21"

m (Solution 2 (Three Variables, Three Equations))
m (Solution 2 (Three Variables, Three Equations))
(8 intermediate revisions by the same user not shown)
Line 15: Line 15:
  
 
==Solution 2 (Three Variables, Three Equations)==
 
==Solution 2 (Three Variables, Three Equations)==
Completing the square in the original equation, we get <cmath>(z-1)\left((z+1)^2+3\right)\left((z+2)^2+2\right)=0,</cmath> from which <math>z=1,-1\pm\sqrt{3}i,-2\pm\sqrt{2}i.</math> Now, we will find the equation of an ellipse <math>\mathcal E</math> that passes through <math>(1,0),\left(-1,\pm\sqrt3\right),</math> and <math>\left(-2,\pm\sqrt2\right)</math> in the <math>xy</math>-plane. By symmetry, the center of <math>\mathcal E</math> must be on the <math>x</math>-axis.
+
Completing the square in the original equation, we have <cmath>(z-1)\left((z+1)^2+3\right)\left((z+2)^2+2\right)=0,</cmath> from which <math>z=1,-1\pm\sqrt{3}i,-2\pm\sqrt{2}i.</math>
  
The formula of <math>\mathcal E</math> is <cmath>\frac{(x-h)^2}{a^2}+\frac{y^2}{b^2}=1,</cmath> with the center at <math>(h,0)</math> and the axes' lengths <math>2a</math> and <math>2b.</math> Plugging the points <math>(1,0),\left(-1,\sqrt3\right),</math> and <math>\left(-2,\sqrt2\right)</math> in, respectively, we get the following system of three equations:
+
Now, we will find the equation of an ellipse <math>\mathcal E</math> that passes through <math>(1,0),\left(-1,\pm\sqrt3\right),</math> and <math>\left(-2,\pm\sqrt2\right)</math> in the <math>xy</math>-plane. By symmetry, the center of <math>\mathcal E</math> must be on the <math>x</math>-axis.
 +
 
 +
The formula of <math>\mathcal E</math> is <cmath>\frac{(x-h)^2}{a^2}+\frac{y^2}{b^2}=1, \hspace{20mm} (*)</cmath> with the center at <math>(h,0)</math> and the axes' lengths <math>2a</math> and <math>2b.</math> Plugging the points <math>(1,0),\left(-1,\sqrt3\right),</math> and <math>\left(-2,\sqrt2\right)</math> into <math>(*),</math> respectively, we have the following system:
 
<cmath>\begin{align*}
 
<cmath>\begin{align*}
 
\frac{(1-h)^2}{a^2}&=1, \\
 
\frac{(1-h)^2}{a^2}&=1, \\
Line 29: Line 31:
 
b^2(-2-h)^2 + 2a^2 &= a^2b^2.
 
b^2(-2-h)^2 + 2a^2 &= a^2b^2.
 
\end{align*}</cmath>
 
\end{align*}</cmath>
Since <math>t^2=(-t)^2</math> for all real numbers <math>t,</math> we rewrite the system as  
+
Since <math>t^2=(-t)^2</math> holds for all real numbers <math>t,</math> we rewrite the system as  
 
<cmath>\begin{align*}
 
<cmath>\begin{align*}
 
(1-h)^2&=a^2, \hspace{24.25mm} &(1)\\
 
(1-h)^2&=a^2, \hspace{24.25mm} &(1)\\
Line 53: Line 55:
 
Substituting the current results into <math>(4),</math> we get <math>b^2=\frac{361}{120}.</math>
 
Substituting the current results into <math>(4),</math> we get <math>b^2=\frac{361}{120}.</math>
  
Finally, we have
+
Finally, we obtain
 
<cmath>c^2 = a^2-b^2 = 361\left(\frac{1}{100}-\frac{1}{120}\right) = \frac{361}{600},</cmath>
 
<cmath>c^2 = a^2-b^2 = 361\left(\frac{1}{100}-\frac{1}{120}\right) = \frac{361}{600},</cmath>
and <cmath>\frac{c}{a}=\sqrt{\frac{c^2}{a^2}}=\sqrt{\frac{361/600}{361/100}}=\sqrt{\frac 16}.</cmath>
+
from which <cmath>\frac{c}{a}=\sqrt{\frac{c^2}{a^2}}=\sqrt{\frac{361/600}{361/100}}=\sqrt{\frac 16}.</cmath>
Our answer is <math>1+6=\boxed{\textbf{(A) } 7}.</math>
+
The answer is <math>1+6=\boxed{\textbf{(A) } 7}.</math>
 +
 
 +
~MRENTHUSIASM
 +
 
 +
==Remark==
 +
The graph of <math>\mathcal E</math> is shown below.
  
Graph of <math>\mathcal E</math> in Desmos: https://www.desmos.com/calculator/ptdpdzsgyo
+
[[File:2021 AMC 12A Problem 21.png|center]]
 +
 
 +
Graph in Desmos: https://www.desmos.com/calculator/ptdpdzsgyo
  
 
~MRENTHUSIASM
 
~MRENTHUSIASM
  
== Video Solution by OmegaLearn (Using Ellipse properties & Quadratic) ==
+
== Video Solution by OmegaLearn (Using Ellipse Properties & Quadratic) ==
 
 
 
https://youtu.be/eIYFQSeIRzM
 
https://youtu.be/eIYFQSeIRzM
  

Revision as of 12:05, 24 April 2021

Problem

The five solutions to the equation\[(z-1)(z^2+2z+4)(z^2+4z+6)=0\] may be written in the form $x_k+y_ki$ for $1\le k\le 5,$ where $x_k$ and $y_k$ are real. Let $\mathcal E$ be the unique ellipse that passes through the points $(x_1,y_1),(x_2,y_2),(x_3,y_3),(x_4,y_4),$ and $(x_5,y_5)$. The eccentricity of $\mathcal E$ can be written in the form $\sqrt{\frac mn}$ where $m$ and $n$ are relatively prime positive integers. What is $m+n$? (Recall that the eccentricity of an ellipse $\mathcal E$ is the ratio $\frac ca$, where $2a$ is the length of the major axis of $\mathcal E$ and $2c$ is the is the distance between its two foci.)

$\textbf{(A) } 7\qquad\textbf{(B) } 9\qquad\textbf{(C) } 11\qquad\textbf{(D) } 13\qquad\textbf{(E) } 15\qquad$

Solution 1

The solutions to this equation are $z = 1$, $z = -1 \pm i\sqrt 3$, and $z = -2\pm i\sqrt 2$. Consider the five points $(1,0)$, $(-1,\pm\sqrt 3)$, and $(-2,\pm\sqrt 2)$; these are the five points which lie on $\mathcal E$. Note that since these five points are symmetric about the $x$-axis, so must $\mathcal E$.

Now let $r:= b/a$ denote the ratio of the length of the minor axis of $\mathcal E$ to the length of its major axis. Remark that if we perform a transformation of the plane which scales every $x$-coordinate by a factor of $r$, $\mathcal E$ is sent to a circle $\mathcal E'$. Thus, the problem is equivalent to finding the value of $r$ such that $(r,0)$, $(-r,\pm\sqrt 3)$, and $(-2r,\pm\sqrt 2)$ all lie on a common circle; equivalently, it suffices to determine the value of $r$ such that the circumcenter of the triangle formed by the points $P_1 = (r,0)$, $P_2 = (-r,\sqrt 3)$, and $P_3 = (-2r,\sqrt 2)$ lies on the $x$-axis.

Recall that the circumcenter of a triangle $ABC$ is the intersection point of the perpendicular bisectors of its three sides. The equations of the perpendicular bisectors of the segments $\overline{P_1P_2}$ and $\overline{P_1P_3}$ are\[y = \tfrac{\sqrt 3}2 + \tfrac{2r}{\sqrt 3}x\qquad\text{and}\qquad y = \tfrac{\sqrt 2}2 + \tfrac{3r}{\sqrt 2}(x + \tfrac r2)\]respectively. These two lines have different slopes for $r\neq 0$, so indeed they will intersect at some point $(x_0,y_0)$; we want $y_0 = 0$. Plugging $y = 0$ into the first equation yields $x = -\tfrac{3}{4r}$, and so plugging $y=0$ into the second equation and simplifying yields\[-\tfrac{1}{3r} = x + \tfrac r2 = -\tfrac{3}{4r} + \tfrac{r}{2}.\]Solving yields $r=\sqrt{\tfrac 56}$.

Finally, recall that the lengths $a$, $b$, and $c$ (where $c$ is the distance between the foci of $\mathcal E$) satisfy $c = \sqrt{a^2 - b^2}$. Thus the eccentricity of $\mathcal E$ is $\tfrac ca = \sqrt{1 - (\tfrac ba)^2} = \sqrt{\tfrac 16}$ and the requested answer is $\boxed{7\textbf{ (A)}}$.

Solution 2 (Three Variables, Three Equations)

Completing the square in the original equation, we have \[(z-1)\left((z+1)^2+3\right)\left((z+2)^2+2\right)=0,\] from which $z=1,-1\pm\sqrt{3}i,-2\pm\sqrt{2}i.$

Now, we will find the equation of an ellipse $\mathcal E$ that passes through $(1,0),\left(-1,\pm\sqrt3\right),$ and $\left(-2,\pm\sqrt2\right)$ in the $xy$-plane. By symmetry, the center of $\mathcal E$ must be on the $x$-axis.

The formula of $\mathcal E$ is \[\frac{(x-h)^2}{a^2}+\frac{y^2}{b^2}=1, \hspace{20mm} (*)\] with the center at $(h,0)$ and the axes' lengths $2a$ and $2b.$ Plugging the points $(1,0),\left(-1,\sqrt3\right),$ and $\left(-2,\sqrt2\right)$ into $(*),$ respectively, we have the following system: \begin{align*} \frac{(1-h)^2}{a^2}&=1, \\ \frac{(-1-h)^2}{a^2}+\frac{{\sqrt3}^2}{b^2}&=1, \\ \frac{(-2-h)^2}{a^2}+\frac{{\sqrt2}^2}{b^2}&=1. \end{align*} Clearing fractions gives \begin{align*} (1-h)^2&=a^2, \\ b^2(-1-h)^2 + 3a^2 &= a^2b^2, \\ b^2(-2-h)^2 + 2a^2 &= a^2b^2. \end{align*} Since $t^2=(-t)^2$ holds for all real numbers $t,$ we rewrite the system as \begin{align*} (1-h)^2&=a^2, \hspace{24.25mm} &(1)\\ b^2(1+h)^2 + 3a^2 &= a^2b^2, &(2)\\ b^2(2+h)^2 + 2a^2 &= a^2b^2. &(3) \end{align*} Applying the Transitive Property to $(2)$ and $(3),$ we get \begin{align*} b^2(1+h)^2 + 3a^2 &= b^2(2+h)^2 + 2a^2 \\ a^2 &= b^2\left((2+h)^2-(1+h)^2\right) \\ a^2 &= b^2(2h+3). \hspace{22.75mm} (4) \end{align*} Applying the results of $(1)$ and $(4)$ to $(2),$ we get \begin{align*} b^2(1+h)^2 + 3\underbrace{b^2(2h+3)}_{\text{by }(4)} &= \underbrace{(1-h)^2}_{\text{by }(1)}b^2 \\ (1+h)^2+3(2h+3)&=(1-h)^2 \\ 1+2h+h^2+6h+9&=1-2h+h^2 \\ 10h&=-9 \\ h&=-\frac{9}{10}. \end{align*} Substituting this into $(1),$ we get $a^2=\frac{361}{100}.$

Substituting the current results into $(4),$ we get $b^2=\frac{361}{120}.$

Finally, we obtain \[c^2 = a^2-b^2 = 361\left(\frac{1}{100}-\frac{1}{120}\right) = \frac{361}{600},\] from which \[\frac{c}{a}=\sqrt{\frac{c^2}{a^2}}=\sqrt{\frac{361/600}{361/100}}=\sqrt{\frac 16}.\] The answer is $1+6=\boxed{\textbf{(A) } 7}.$

~MRENTHUSIASM

Remark

The graph of $\mathcal E$ is shown below.

Graph in Desmos: https://www.desmos.com/calculator/ptdpdzsgyo

~MRENTHUSIASM

Video Solution by OmegaLearn (Using Ellipse Properties & Quadratic)

https://youtu.be/eIYFQSeIRzM

~ pi_is_3.14

See also

2021 AMC 12A (ProblemsAnswer KeyResources)
Preceded by
Problem 20
Followed by
Problem 22
1 2 3 4 5 6 7 8 9 10 11 12 13 14 15 16 17 18 19 20 21 22 23 24 25
All AMC 12 Problems and Solutions

The problems on this page are copyrighted by the Mathematical Association of America's American Mathematics Competitions. AMC logo.png